LSAT and Law School Admissions Forum

Get expert LSAT preparation and law school admissions advice from PowerScore Test Preparation.

User avatar
 Dave Killoran
PowerScore Staff
  • PowerScore Staff
  • Posts: 5853
  • Joined: Mar 25, 2011
|
#43438
Complete Question Explanation
(The complete setup for this game can be found here: lsat/viewtopic.php?t=16399)

The correct answer choice is (B)

Answer choice (A) is incorrect because N and T cannot be selected together.

Answer choice (B) is incorrect because F and H cannot be selected together.

Answer choice (C) is incorrect because if K is selected then N must be selected.

Answer choice (D) is the correct answer choice.

Answer choice (E) is incorrect because if H is selected then K must be selected.

Get the most out of your LSAT Prep Plus subscription.

Analyze and track your performance with our Testing and Analytics Package.